LSAT and Law School Admissions Forum

Get expert LSAT preparation and law school admissions advice from PowerScore Test Preparation.

 norvo
  • Posts: 2
  • Joined: Nov 11, 2014
|
#17340
Good Afternoon,

Can I please receive some explanation regarding question 16 as part of the Logical Reasoning Problem Set #1.

#16: "Though they soon will, patients... ." Can you please explain the question stem and also why A is correct and not E.


Thank you for your help!
 David Boyle
PowerScore Staff
  • PowerScore Staff
  • Posts: 836
  • Joined: Jun 07, 2013
|
#17353
norvo wrote:Good Afternoon,

Can I please receive some explanation regarding question 16 as part of the Logical Reasoning Problem Set #1.

#16: "Though they soon will, patients... ." Can you please explain the question stem and also why A is correct and not E.


Thank you for your help!
Hello norvo,

We have a resolve-the-paradox thing here. Patients will waste time by asking for records...but none of them are going to ask for the records! Huh??
Answer choice A solves this by saying that there'll be a legal duty to have the records ready, even if nobody asks for them. Answer E may strengthen the second part of the paradox, by implying that patients may not ask for their records; but that does not resolve the paradox.

Hope this helps,
David

Get the most out of your LSAT Prep Plus subscription.

Analyze and track your performance with our Testing and Analytics Package.